Re: Gruppi ciclici

Messaggioda algibro » 21/02/2018, 08:01

Shocker ha scritto:
algibro ha scritto:Riscrivo la dimostrazione provando un'altra strada :D .
Così, siccome $ m $ è per ipotesi un intero positivo, ci sono due possibilità per $ r $:
1) $ r=h $ ma ciò contraddice $ r < h $ dalla divisione euclidea;
2) $ r=0 $ da cui $ g^k = g^{hq} = (g^h)^q \in H $

Non ho capito queste tre righe.


Mi sa che farneticavo... :(

Shocker ha scritto:
algibro ha scritto:E poi scusate, ma se $ h < k $ e $ hm, km $ sono due periodi di $ g $, deve essere $ hm|km \Rightarrow km=hmq \Rightarrow k=hq $, no ?

Detta così è falsa: prendi $g$ di ordine $6$. Allora $g^12 = g^18 = e$ ma $12$ non divide $18$.


Maledizione ! Ciò che ho scritto è vero solamente se il primo dei due periodi è anche l'ordine di $h$ !!!
Più in generale se $o(a)=n$ e $a^k=e$ allora $n|k$ (1).
Nel nostro caso abbiamo che $o(g)=n$ e $g^{hm}=h^{km}=e$. Allora $n|hm$ e $n|km$.


Shocker ha scritto:
algibro ha scritto: $n | mk - mh = m(h-k) \Rightarrow $scusate da qui in poi mi sa che ho scritto una cavolata, adesso ci penso su $ h \equiv k (mod n) $

In generale questo è falso, quello che puoi dire è che $h \equiv k mod (\frac{n}{(n, m)})$, in questo caso $(n, m) = m$ perché $m | n$.


Ok, vediamo se ho capito perché $m|n$.
Per le ipotesi $o(g^k)=m$. Inoltre $e=g^n=(g^n)^k=g^{nk}=(g^k)^n$. Allora per quanto detto in (1) abbiamo che $m|n$.

Shocker ha scritto:Comunque ci siamo quasi: se $k \equiv h \equiv 0 \mod(\frac{n}{m})$(è chiaro perché è zero, no?) allora sono entrambi multipli di $\frac{n}{m}$. Sarebbe interessante scoprire che ordine ha $g^\frac{n}{m}$.


Se $g^{km}=e=g^0$. Allora $km \equiv 0 (mod n) \Rightarrow km=nq \Rightarrow k=\frac{n}{m}q \Rightarrow k \equiv 0 (mod \frac{n}{m})$.
Stesso discorso per $h$, d'altra parte se $hm, km$ sono due potenze che applicate a $g$ restituiscono l'elemento neutro, allora entrambe saranno nella classe di resto zero modulo l'ordine di $G$.
L'ordine di $g^\frac{n}{m}$ è $m$, infatti $(g^\frac{n}{m})^m=e$.
In conclusione posso considerare $\langle$ $g^\frac{n}{m}$ $\rangle$, il sottogruppo di $G$ generato da $g^\frac{n}{m}$.
Ora, ogni potenza di $(g^h)$ e di $(g^k)$ è della forma $(g^\frac{n}{m})^t$ per $t \in ZZ$, dunque
$H=\langle$ $g^\frac{n}{m}$ $\rangle = K$

Spero più o meno di esserci, sono argomenti che si affrontano facilmente, ma altrettanto facilmente mi ci perdo.
algibro
Junior Member
Junior Member
 
Messaggio: 154 di 378
Iscritto il: 29/01/2017, 15:16

Re: Gruppi ciclici

Messaggioda Shocker » 21/02/2018, 11:26

Allora diciamo che ci sei ma devi essere più preciso su un punto.

algibro ha scritto:Ok, vediamo se ho capito perché $ m|n $.
Per le ipotesi $ o(g^k)=m $. Inoltre $ e=g^n=(g^n)^k=g^{nk}=(g^k)^n $. Allora per quanto detto in (1) abbiamo che $ m|n $.

Mi sembra ok. Alternativamente puoi dire che $g^k$ genera un sottogruppo di ordine $m$ e quindi per il teorema di lagrange $o(g) = |H| | |G| = n$.


L'ordine di $ g^\frac{n}{m} $ è $ m $, infatti $ (g^\frac{n}{m})^m=e $.

Ecco qui devi essere più preciso: in questo modo hai dimostrato che $o(g^{\frac{n}{m}})| m$ ma a te serve provare che $m = o(g^\frac{n}{m})$.
#NikkioAlleIMO - https://www.youtube.com/watch?v=vEl5bFIALb8

"Se vivessimo in $\mathbb{R^4}$ allora nessuno si impiccherebbe perché in $\mathbb{R^4}$ tutti i nodi si sciolgono"
Avatar utente
Shocker
Senior Member
Senior Member
 
Messaggio: 803 di 1738
Iscritto il: 02/10/2011, 17:09

Re: Gruppi ciclici

Messaggioda algibro » 23/02/2018, 20:21

Shocker ha scritto:
algibro ha scritto:L'ordine di $ g^\frac{n}{m} $ è $ m $, infatti $ (g^\frac{n}{m})^m=e $.

Ecco qui devi essere più preciso: in questo modo hai dimostrato che $o(g^{\frac{n}{m}})| m$ ma a te serve provare che $m = o(g^\frac{n}{m})$.


Bene, allora supponiamo per assurdo l'esistenza di $t \in NN$, strettamente minore di $m$, che sia il più piccolo intero positivo tale che $(g^\frac{n}{m})^t=e$. Così si avrebbe $g^{n\frac{t}{m}}=e$. Poiché $m|n, m <=n$ e per ipotesi $t<m$ allora $n\frac{t}{m} < n$ e ciò contraddice l'iniziale scelta di $n$ quale ordine di $g$.
Per completezza, non riscontro problemi con le frazioni in quanto se $m|n$ allora $n\frac{t}{m}$ è un intero.
algibro
Junior Member
Junior Member
 
Messaggio: 155 di 378
Iscritto il: 29/01/2017, 15:16

Re: Gruppi ciclici

Messaggioda anto_zoolander » 22/04/2018, 15:00

Mi sono decisamente dimenticato di rispondere al mio stesso post :weedman: a questo punto completo.

Sia $d| o(G):=n$ allora $n=dk$ e il sottogruppo $<g^k>$ ha ordine $d$ di fatto

$(g^k)^d=g^(kd)=g^n=e$


quindi $d$ è un periodo, ovviamente è anche il più piccolo visto che se $h<d$ allora $hk<dk=n$ ovvero $g^(kh)ne e$ da cui $(g^k)^h ne e$ pertanto $o(g^k)=d$ e segue che $<g^k>$ è un sottogruppo di ordine $d$

è anche unico, di fatto, supponiamo che esista $HleqG$ di ordine $d$ e poichè $H$ è sottogruppo di un gruppo ciclico si avrà che $H$ è ciclico ovvero $H= <g^m>$

ora $m*d$ divide $n$ dividendo $md=nq+r$ si ottiene subito $r=0$
da questo $m*d=n*q=(dk)*q=(kq)*d => d(m-kq)=0$ ma $d ne0 => m=kq$

quindi $g^m=(g^k)^q$ segue che $Hsubseteq <g^k>$ ed hanno lo stesso ordine, quindi coincidono.
Error 404
Avatar utente
anto_zoolander
Moderatore
Moderatore
 
Messaggio: 2327 di 9002
Iscritto il: 06/10/2014, 15:07
Località: Palermo

Precedente

Torna a Algebra, logica, teoria dei numeri e matematica discreta

Chi c’è in linea

Visitano il forum: Nessuno e 1 ospite